0% found this document useful (0 votes)
557 views160 pages

TOEFL

Uploaded by

Muhammad Alfatih
Copyright
© © All Rights Reserved
We take content rights seriously. If you suspect this is your content, claim it here.
Available Formats
Download as PDF or read online on Scribd
0% found this document useful (0 votes)
557 views160 pages

TOEFL

Uploaded by

Muhammad Alfatih
Copyright
© © All Rights Reserved
We take content rights seriously. If you suspect this is your content, claim it here.
Available Formats
Download as PDF or read online on Scribd
You are on page 1/ 160
LISTENING COMPREHENSION ‘The first section of the TOEFL test is the Listening Comprehension section. This section consists of fifty questions (some tests may be longer). You will listen to recorded materials and respond to questions about the material. You must listen carefully, because you will hear the recording one time only, and the material on the recording is not written in your test book. ‘There are three parts in the Listening Coriprehension section of the ‘TOEFL test: 1, Part A consists of thirty short conversations, cach followed by a question. ‘You must choose the best answer to each question from the four choices in your test book. 2. Part B consists of two longer conversations, each followed by a number of questions. You must choose the best answer to each question from the four choices in your test book. 7 Part C consists of three talks, each followed by a number of questions. You must choose the best answer to each question from the four choices in your test book. GENERAL STRATEGIES 1. Be familiar with the directions. The directions on every TOEFL test are the same, so it is not necessary to listen carefully to them each time. You should be completely familiar with the directions before the day of the test. . Listen carefully to the conversations and talks. You should conentrate fully on what the speakers are saying on the recording because you will hear the recording ‘one time only. . Know where the easier and more difficult questions are generally found. Within teach part of the Listening Comprehension section, the questions generally progress from easy to difficult. . Never leave any questions blank on your answer sheet. Even if you are unsure of the correct response, you should answer the question. There is no penalty for guessing. . Use any remaining time to look ahead at the answers to the questions that follow. When you finish with one question, you may have time to look ahead at the answers to the next question. 10 _USTENING COMPREHENSION. THE LISTENING PART A QUESTIONS For each of the thirty questions in Part A of the Listening Comprehension section of the TOEFL test, you will hear a short conversation between two speakers fol- lowed by a question. The conversations and questions are not written in your test ‘book. After you listen to’the conversation and question, you must choose the best answer to the question from your test book. ‘Example On the recording, you hear: (man) I need a car to drive to Arizona, and I don't have one. (woman) Why not rent one? (narrator) What does the woman suggest? In your test book, you read: (@) Getting a red car (B) Renting an apartment in Arizona, (©) Not driving to Arizona. (D) Renting a car for the trip. ‘Answer (D) is the best answer to the question. Why not rent one? is a suggestion that he rent a car for his trip to Arizona. STRATEGIES FOR THE LISTENING PART A QUESTIONS 1. As you listen to each short conversation, focus on the second line of the conversation. The answer to the question is generally found in the second line of the conversation. 2. Keep in mind that the correct answer is probably a restatement of a key word or Idea in the second line of the conversatioh. Think of possible restatements of the second line ofthe conversation. 3. Keep in mind that certain structures and expressions are tested fegularly in Listening Part A. Listen for these structures and expressions: + restatements i + negatives i + suggestions: + passives + conclusions about who and where + expressions of agreement 4, Keep in mind that these questions generally progress from easy to difficult. This ‘means that questions 1 through § will be the easiest questions and questions 26 through 30 will be the hardest questions. USTENING PARTA 11 5. Read the answers and choose the best answer to each question. Remember to ‘answer each question even if you are not sure of the correct response. Never leave any answers blank. 6. Even If you do not understand the complete conversation, you can find the correct answer. + Ifyou only understood a few words or ideas in the second line, choose the answer that contains a restatement of those words or ideas. + If you did not understand anything at all in the second line of the conversation, choose the answer that sounds the most different from what you heard. + Never choose an answer because it sounds ike what you heard in the conversation. The following skills will help you to implement these strategies in Part A of the Listening Comprehension section of the TOEFL test. Skt 1: RESTATEMENTS. Often the correct answer in Listening Part A is an answer that contains a restate- ment of the ideas in the second line of the conversation. Example On the recording, you hear: (woman) Steve, is something the matter? You dont look very ‘ood. (aman) Oh, I'm feeling a litle sick today. (narrator) What does the man mean? Tn your test book, you read: (A) He's not very good-looking. (B) Hes a bt il (©) He looks worse than he feels. (D) His fet are alittle thick. In this conversation, sick means il, and a little means a bit. The best answer to this question is therefore answer (B) because answer (B) restates the idea in the second line of the dialogue. Please note that answer (D) is definitely not a correct answer because it contains feet instead of feel and thick instead of sick. These ‘words are similar in sound but not in meaning to the words that you hear on the recording. a LUSTENING COMPREHENSION. ‘The following chart outlines the most important strategy in Listening Part A. [NOTE In Append A there are dls to practice elstnguishing Similar sounds. You may want to complete these practice dlls before trying the folowing exercises. EXERCISE 1: In this exercise, underline the key idea(s) in the second line of each short conversation. Then underline restatements of these key words in the ‘answers, and choose the best answer to each question. Remember that the best answer is probably the answer that contains a restatement of the key idea(s) in the second line of the conversation. 1. (woman) What do you like about your (A) The house is closed up now. now house? (B) He parks his car close to his house. (man) Its very clase t0 a park. (C) His home is near a park. (narrator) What does the man mean? (D) He doesn't really like his new house. 2, (man) Mark said some really nice (A) He did a great job. things to me, (B) He bought a crate full of fruit. (woman) He's very grateful for what (C) He made a great fool of himself. you did, (D) He’ thankful. (narrator) What does the woman say ‘about Mark? 3. (woman) Sam, you've been working at (A) ‘The computer’s broken. the computer for hours. (B) He needs to relax for a while. (man) Yes, and [need to take a (C) He should keep working. break. (D) He's broke. (narrator) What does the man mean? TOEFL EXERCISE 1: In this exercise, USTENING PART A 13 isten carefully to the short conversation and question on the recording, and then choose the best answer to the question. ‘You should look for restatements of key ideas in the second line. (A> iow scan 1 rcononc at TOEFL Bxnase 1. 1. (A) The dance was fun. (B) It was a good time to dance. (©) She thinks the man is such a ‘good dancer. (D) Tonight is the last time to dance. 2. (A) She's quite thirsty. (B) She'll be ready in a half hour (©) She needs to leave now. (D) She was ready thirty minutes ago. 3. (A) The woman should teach the class herself, (B) The woman should see a sychiatrist. ‘good idea to speak with the instructor (D) He would like to discuss psychology with the woman. 4, (A) She has a starring role. (B) She has not found a job yet. (© She found the lost dog. (D) She just began working. 5, (A). He shares everything he has. (B) He has to find his lost shirt. (© He is climbing the stairs to the apartment. (D) He has to get a roommate. 6. (A) His vacation is full of activities. (B) In autumn he'll have a vacation. (©) He can't have a vacation because he suffered a fall. (D) He's foolish to take a vacation now. 7. (A) The book was very enjoyable. (B) Itwas unpleasant to get rid of the bugs. (©) She only reads when itis quiet. (D) Itis pleasant to ride a bike. 8, (A) He hasnt traveled much lately. (B) There were a lot of cars on the road. (©) He needs to lose weight. (D) The tray was thick and heavy. 9. (A). He doesn't know how to play the game. (B) He has to write a story for class. (©) He needs to complete an. assignment. (D) He has already finished the report. 10. (A) He always minds his own. hhusiness, (B) He manages to avoid working, (C) He is the manager of the department. (D) Hes the boss of his own ‘company. 14 _USTENING COMPREHENSION Skit 2: NEGATIVES ——__————————————————— Negative expressions are very common in Listening Part A. The most common Kind of correct response to a negative statement is a positive statement containing a word with an opposite meaning. Example On the recording, you hear: (woman) Did you get a lot of work done at the library today? (man) Icouldnt. It wasn't very quiet there, (narrator) What does the man mean? In your test book, you read: (A) The library was noisy. (B) He got a lot done. (©) He couldnt quite get to the library today. (D) The library’ a good place to work because it's quiet. ‘The correct answer is answer (A). If it was not quiet in the library, this means that it was noisy. Notice that the correct answer uses noisy, the opposite of quiet. ‘The following chart shows how negatives can be tested on the TOEFL test. EXERCISE 2: In this exercise, underline the negative expression in the second line of each short conversation. Then read the question, and choose the best answer to that question. Remember that the best answer is one that uses an oppo- site meaning. 1, (woman) You made so many mistakes (A) He was rather careless. in this homework. (B) He does not care about mistakes. (man) Iwasn' very careful. (©) He took care of the work at home. (narrator) What does the man mean? _(D) He did not carry the work home. 2, (man) Is there a lot of soup? Im kind of hungry. (woman) Sorry, there's not a lot: (narrator) What does the woman mean? 3. (woman) How was the weather on ‘your trip? (man) There wasn't a cloud in the sh. (narrator) What does the man mean? USTENING PARTA 15 (A) There's not very much soap. (B) She doesnt like soup. (©) There’ only a litle soup. (D) The man should not be hungry. (A) Ie was cloudy. (B) He couldn't see the sky. (©) It wasn't very crowded. (D) There was a lot of sunshine. TOEFL EXERCISE 2: In this exercise, listen carefully to the short conversation and question on the recording, and then choose the best answer to the question. ‘You should pay particular attention to negative expressions. OY niow scan nie seconons ar TOEFL Exerase 2. 1. (A) He was on time. (B) He's sorry he was late. (C) He doesn't know why he was late, (D) He hasn't come there lately. 2, (A) She's sorry she brought the book. (B) She remembered to say she was sony. (©) She forgot the book. (D) She didn't remember the ring, 3. (A) The baby’s nodding off. (B) The baby’s asleep. (C) The baby’s on the way home. (D) The baby's just waking up. 4, (A). He wants to go by himself. (B) He isn't going on the trp. (©) He has a large number of friends. (D) He isn't traveling alone, 5. (A) She does not have time to park the car. (B) She has some free time. (C) She has not been to the park in a while. (D) The park is too wild. 6. (A) He closed the windows. (B) Ie didn't rain. (©) The windows were open during the storm. (D) He saw the rain through the ‘windows. 7. (A) Shes certain the plant’ alive. (B) She's not sure what happened to the plant. (©) She planned to throw it out, (D) She's sure the plant should be thrown out. 8. (A) The apple is good because it’s sweet, (B) There is an apple in each suite. (C) The apple doesn't look good. (D) The apple is sour. 9. (A) It’s cheap. (B) Itcostsa lot. (©) Ithas a few problems. (D) Its hard to find. 410, (A) She must leave the rock there. (B) She needs a big rock. (©) She should take the rock with her. (D) She should take a bigger rock. 16 LUSTENING COMPREHENSION. TOEFL REVIEW EXERCISE (Skills 1-7 In this exercise, listen carefully to the short conversation and question on the recording, and then choose the best answer to the question. Cie nicc tote cas 1. (A) She couldnt think ofa single answer. (B) The test was easy. (©) Itwas impossible to think during the exam. (D) Itwas too quiet. 2. (A), He will finish quickly. (B) He works slowly. (C) He isn't worried. (D) He doesn't like to work. 3. (A) Her mother and father were hungry. (B) She was angry at her parents. (©) Her mother and father got home too late. (D) Heer parents were mad. 4, (A) He didnt get a car (B) The car he got wasnt real; it was toy. (©) He really wanted a car but couldn't get one. (D) The car that he just bought is old. 5. (A) Take some seeds. (B) Sit down. (©) Make an appointment. (D) Take some'time. ‘Skit 3: SUGGESTIONS. 6. (A) Itwas difficult to play because of the wind. (B) Hes unhappy because they lost. (©) He doesn't like playing football in the winter. (D) Hes not very happy about the ‘way they won, 7. (A). He paid the rent two days ago. (B) The rent wasn't due the day before yesterday. (©) One day earlier he'd rented an apartment. (D) Yesterday he spent the whole day ranting. 8. (A) Barry’ too old to enjoy camping. (B) Not enough people are going camping. (© This weekend is not a good time for camping. (D) Barry’ too young to go camping. 9. (A) He has to water the lawn, (B) He's sorry about having to move. (C) He's washing his clothes. (D) He can't move right now. 10. (A) He thinks the weather there is great. (B) He prefers dry weather, (©) The weather there is good for humans. (D) He likes wet weather ‘Suggestions are common in Listening Part A, so you should become familiar with them. The following example shows an expression of a suggestion. USTENING PARTA 17 ‘Example On the recording, you hear: (man) I havent talked with my parents in a while. (woman) Why dont you call them now? (narrator) What does the woman suggest? Tn your test book, you read: (A) Calling off his visit. (B) Talking about his parents, (©) Calling his parents in a while (D) Phoning his family. In this example, the expression why dont is an expression of suggestion, so the ‘woman's suggestion is to call them. This means that the woman suggests phoning his family. The best answer is therefore answer (D). ‘The following chart lists common expressions that show suggestion. EXERCISE 3: In this exercise, underline the expression of suggestion in each short conversation. Then read the question, and choose the best answer to that ‘question. Remember that the best answer is one that gives a suggestion. 1. (man) The weathers so beautiful (A). Taking the car to school, today. (B) Taking a walk instead of going to (woman) Let's walk 10 school instead school. of driving. (©) Going for a drive in the beautiful (narrator) What does the woman ‘weather. ‘suggest? (D) Walking to clas. 2. (woman) Inever have enough (A). Spending less money. spending money. (B) Doing a better job at work. (man) Why not get a part-time job? (C) Earning some money. (narrator) What does the man suggest? (D) Spending less time at her job. 3. (man) We don't have any plans (A) Having a dinner party. tonight, do we? (B) Stopping the fight with their friends. (woman) No, so why don't we invite (C) Planninga trip. some friends over to (D) Making some new friends. dinner? (narrator) What does the woman ‘suggest? LUSTENING COMPREHENSION TOEFL EXERCISE 3: In this exercise, listen carefully to the short conversation and question on the recording, and then choose the best answer to the question. ‘You should pay particular attention to expressions of suggestion. (G) sew steve rcs 108 Cane 3 1. (A) Going to work. (B) Getting some exercise. (©) Relaxing for a while (@D) Visiting her friend Jim, 2. (A) Completing the work later. (B) Finishing more of the work now. (©) Trying to do the work today. (D) Resting tomorrow. 3. (A) Cooling off the house. (B) Turning down the heat. (©) Raising the temperature. (D) Finding something to eat. 4. (A) Fixing the car (B) Breaking the news to the others. (© Buying another automobile. (©) Going down the hill in the car. 5. (A) Putting up shelves. (B) Selling the books. (©) Looking for the lost books. (D) Rearranging the books. 6. (A) Visiting her sister. (B) Getting a baby-stter. (©) Letting her sister see the clothes. () Giving the clothes away. 7. (A) Postponing the trip. (B) Leaving immediately. (©) Putting on a play tomorrow. (D) Going later tonight. 8. (A) Working on the term paper the whole day. (B) Spending a little money on paper (©) Doing a beter job of planning her term paper (©) Spending the whole term on the paper 9. (A) Getting a new television. (B) Checking which programs are coming on. (©) Checking their watches, (D) Seeing a comedy on television. 10. (A) Making some extra money. (B) Depositing the money in his account. (©) Buying something with the ‘money. (D) Checking out several banks. TOEFL REVIEW EXERCISE (Skills 1-3): In this exercise, listen carefully to the short conversation and question on the recording, and then choose the best answer to the question. Cy ees geet ee reer 1. (A) He prefers to stay home. (B) He has to take many trips. (© He has alot of tasks to do. (D) He works well at home. 2, (A) There's nothing in the car. (B) He has enough time now. (C) He doesn't want a car now. (D) He prefers to do it later. 3, (A). Making less noise. (B) Spending more time away from the apartment. (© Moving the furniture around the apartment. (D) Changing apartments. 4. (A). He overstepped his authority. (B) He worked too much in the ‘morning. (©) He stayed in bed too long. (D) He was late getting home in the ‘morning, 5. (A) She's rather quiet. (B) He doesnt really know her. (©) She's had many difficulties. (D) He's never talked to her 6. (A) Fixing something to drink. (B) Adding lemon to the cake. (©) Paying thirty dollars. (D) Picking lemons from the tree. LUSTENING PART A 7. (A) Write a letter: (B) Complete the assignment for him, (©) Tell him the time. (D) Phone him in a while. 8, (A) Getting up in the afternoon. (B) Having his hair cut. (©) Cutting the flowers in the garden. (D) Growing his hair long. 9. (A) She's sure she saw him. (B) She doubts what he said. (© She isn't sure she told him her name. (D) She shares his beliefs. 10, (A) ‘The break regularly lasts an hour. (B) No dishes were broken. (C) The break comes at its usual time. (D) It regularly takes an hour to rake the lawn. ‘Skit 4: PASSIVES —————_—_$_$_$_$__ It is sometimes difficult to understand whio or what is doing the action in a passive ‘sentence. This problem is often tested in Listening Part A. Example On the recording, you hear: (man) Is that a new chair? In your test book, you read: (woman) Yes, we just bought it last week. (narrator) What does the woman mean? (A) She brought the chair with her. (B) The chair was lost for a week. (C) The chair was purchased recently. (D) She bought the last chair from the store. In this conversation, the woman uses an active idea, we... bought it [tHe chair). ‘The correct answer uses the passive idea the chair was purchased. Therefore, the best answer to the question above is answer (C) 0 20 _USTENING COMPREHENSION You should note the following about passive sentences in Listening Part A. EXERCISE 4: In this exercise, each of the correct answers is either a passive restatement of an active sentence or an active restatement of a passive sentence. Read the short conversation, and underline the key active or passive statement. Then read the question, and choose the best answer to the question. Make sure you understand who.or what is doing the action in these passive sentences. 1, (man) What happened to your (A) The cafeteria isto the left. notebook? (B) She let a note on the cafeteria door. (woman) 1 ft it in the cafeteria, (©) She took some notes inthe cafeteria. (narrator), What does the woman () The notebook was left in the imply? cafeteria E 2 (man) When are we going to talk (A) She needs to make copies before the | about the problem with meeting. | the copy machine? (B) They will talk about the problem “) (woman) Ze will be discussed at the tomorrow. ‘meeting tomorrow. (©) Itisa problem to have a meeting. | (narrator) What does the woman (©) They must discuss whether oF not to ‘mean? havea meeting. 3. (woman) Did you correct the paper? _(A) "The paper has been thoroughly (man) I checked every single line corrected. iy again and again. (B) Hee wrote many of the lines over and (narrator) What does the man mean? over (©) The checkmarks were on every line of the paper, (D) He wrote a check for the paper, USTENING PART A 21 TOEFL EXERCISE 4: In this exercise, listen carefully to the short conversation land question on the recording, and then choose the best answer to the question, ‘You should pay particular attention to passives. A> om sev necorons 1 TOEFL Exc 4 1, (A) Her family just arrived, 6. (A) Nothing is really the matter. (B) She must pack to go visit her (B) She stole some money. family. (©) She left her purse in the store. (© She opened the door to greet her (D) Someone took her purse family. (D) The box from her family was 7. (A) Hes quite scared. ‘opened. (B) He likes her pets. (©) He would like to pick up a little 2. (A) He completed the report. snake. (B) Hes coming to work on the (D) He frightens the snakes. report. (©) The report is due tomorrow 8. (A) She wrote directions in a letter. ‘moming. (B) Instructions were followed (D) The report still needs an infinite ‘exactly. amount of work. (©) Anew director has been appointed. 3. (A) The parents are in bed now. (D) She would follow the man to the (B) The children were allowed to stay housing project. up. (©) The parents stayed away from 9. (A) They can cook supper in no time. the children. (B) He doesn't know how to play (D) The children have gone to bed. tennis. (© He's a super cook. 4, (A) He got lost. (D) Dinner needs to be prepared. (B) The door was not in front of the house 10, (A) The exam really is tomorrow. (©) He did not need a key to the (B) There was a change in the door. content of the exam. (D) The key was lost. 5, (A) He picked out some flowered ‘wallpaper for the dining room. (B) The dining room table has flowers painted on it. (©) The flowers were placed on the table. (D) The flowers were grown in the dining room. (©) The professor moved the exam to another day. (D) They have to change their plans for tomorrow. 2 LUSTENING COMPREHENSION TOEFL REVIEW EXERCISE (Skills 1-4); In this exercise, listen carefully to the short conversation and question on the recording, and then choose the best answer to the question, C7 Wo scr recon TOEFL Revew Eense (as 1-4) 1. (A) It's the middle of the winter. (B) The weather is not very calm. (C) The boat needs wind to go. (D) They need to unwind the sail 2. (A) Taking two hats with him, (B) Aspiring to greater achievements, (C) Taking some medication. (D) Moving his head. . (A) He gets up every day at sunrise. (B) The door is open to let the sunshine in, (©) He parks his car out of the sunlight. (D) The park is open all day long, . (A) She ate part of Steve's meal (B) Steve didn't pay for his meal. (©) Steve had five courses for dinner. (D) She was given somie money. 5. (A) He did the problem completely. (B) He corrected the homework. (© He was totally wrong. (D) He did well on the math exam. (A) The washing machine was repaired. (B) He washed some clothes this ‘morning. (C) He broke some dishes while he ‘was washing them. (D) The washing machine needs to be fixed. 2 10. (A) Registering in algebra ‘immediately. (B) Finding another school. (C) Enrolling in the course later. (D) Repeating the course next ‘semester. (A) les red, (B) Irs not dry. (© Itneeds to be watched, (D) He's going to wash it. (A) Everyone feels relaxed at the party. (B) There are enough people to have a party. (©) The amount of food is adequate. (D) Part of the food is on the table, (A) The meeting was at four o'clock. (B) Everyone came to the meeting in ‘uniforms. (©) The information was given at the (©) No one was uninformed about the meeting. USTENING PART A 23 WHO AND WHERE —————_________—__ tis common in Listening Part A to ask you to draw some kind of conclusion. In this type of question the answer is not clearly stated; instead you must draw a conclusion based on clues given in the conversation. One kind of conclusion that is common in this part of the test is to ask you to determine who the speaker is, based on clues given in the conversation. ‘SKILL Example ‘On the recording, you hear: (aman) What do you do during your performances? (woman) I play the piano and sing. (Garrator) Who isthe woman most likely to be? In your test book, you read: (A) Anathlete. (B) Amember of the audience, (©) Aclerk in a music store, (©) A musician, The clues performances, piano, and sing in the conversation tell you that the ‘woman is probably a musician. Answer (D) is therefore the correct answer. ‘Another type of conclusion that is common in Listening Part A is to determine where the conversation probably takes place, based on clues given in the conver- sation. Example On the recording, you hear: (man) 1 like to open an account, please (woman) What type of account would you like, a checking or savings account? (narrator) Where does this conversation probably take place? In your test book, you read: (A) Inan accounting class. (B) Ina bank. (©) Ata service station, (D) Ina market, ‘The clue checking or savings account in the conversation tells you that this con- versation probably takes place in a bank. Answer (B) is therefore the correct m LUSTENING COMPREHENSION The following chart outlines the key point that you should remember about this type of question. EXERCISE 5: In this exercise, read the short conversation and question, under- line the clues that help you answer the ‘question, and then choose the best answer. ‘You will have to draw conclusions about who and where. 1. (man). like to mail this package, lease. (woman) First or second class? (narrator) Who is the woman most likely to be? 2. (woman) How much are the flowers? (man) Ten dollars a bouquet, (narrator) Where does this conversation probably take place? 3. (man) I need 10 get this prescription filled. (woman) If you'll take a seat, IM have ‘your medicine ready for ‘you in a moment. (narrator) Who is the woman most ely to be? (A) A school administrator. (B) A postal worker. (© Abanker (D) A teacher, (A) Ina bakery (B)_In a grocery store. (©) Ina florist shop. (D) Ina garden, (A) A pharmacist. (B) A baby-sitter (©) A flight attendant. (D) A doctor. TOEFL EXERCISE 5: In this exercise, listen carefully to the short conversation and question on the recording, and then choose the best answer to the question, ‘You will have to draw conclusions about who and where. (CO) iow sxc nconons at TOEFL Een 5, 1. (A) Ina hotel room. (B) Ata restaurant. (©) Atthe beach. (D) Ina desert. 2. (A) Amanager, (B) A teacher. (©) Alawyer (D) Alibrarian. 3. (A) Ina library. (B) Ina doctor's office. (© Imabookstore. —* (D) Ina grocery store. 4, (A) Abeautician. (B) A secretary. (© Adentist. (D) A gas station attendant. 5. (A) Ina plane. (B) Inacar (© Onabus. () Ona boat. 6. (A) A weather forecaster. (B) A minister. (C) A marriage counselor. (D) A bride. LUSTENING PART A: 7. (A) In an airport. (B) Ata gas station. (© Ata supermarket. (D) Ima bike shop. 8. (A) A musician. (B) An office worker. (© Aprofessor. (D) Anathlete. 9. (A) Ina clothing store. (B) Ata carwash, (© Ina laundry. (D) Ata grocery store. 10. (A). A bank teller (B) A travel agent. (©) Apolice officer (D) Abus driver. ‘TOEFL REVIEW EXERCISE (Skills 1-5): In this exercise, listen carefully to the short conversation and question on the recording, and then choose the best answer to the question. O7 niow vx recon ar TOEFL Revew Bence (Ses 1-5). 1. (A) Shutting the window. (B) Moving closer to the window. (©) Closing down the building, (D) Cooling the room off. 2. (A) The plane came in at night. (B) The “ight was scheduled to last ‘two hows, (C) The fight started at two o'clock. (D) The plane landed on time 3. (A) Ina clothing store. (B) Ata pool. (©) Ata racetrack. (D) Ina restaurant. 4, (A) He's unhappy with the sofa. (B) He likes the news. (C) He's heard the unpleasant news. (D) He always tries to please everyone. 55, (A) He missed the bus he wanted to take. (B) He's sorry he bought the yellow shirt. (©) He made a huge error (D) What happened was a mystery. 6. (A) Tuition is due soon. (B) She has a free day tomorrow. (C) The school is free. (D) The room will be painted tomorrow. 7. (A) He plays football regularly. (B) He heard some people playing football. (©) He was injured during a game. (D) He hurt someone during a game. (continued on next page) 25 26 —_USTENING COMPREHENSION 8. (A) Some of the seats were empty. 9. (A) Putting on a new sult. (B) All the chairs were full. (B) Tasting the soup again. (©) The lecturer was standing the (©) Putting some soup in the bowls. whole time. (D) Adding flavorings to the soup. (D) He was seated at the back of the lecture hal 10. (A) A student. {(B) A receptionist. (©) Asalesclerk. () Adean. ‘Skit 6: AGREEMENT. Expressions of agreement are common in Listening Part A, s0 you should become familiar with them, The following example shows agreement with a positive state- ‘ment. Example ‘On the recording, you hear; (man) I thought that the meal was overpriced. (woman) too. faery ne ey ene In your test book, you read: (A) There were too many spices in the meat. (B) She has the same opinion of the meal as the man. (©) She wants to share the man's meal. (D) The price of the meal was great. ‘The expression me, too shows agreement with a positive statement, so the woman means that she has the same opinion as the man. The best answer is therefore answer (B). ‘The following chart lists common expressions that show agreement. You should become familiar with these expressions. USTENING PART A 27 EXERCISE 6: In this exercise, underline the expression of agreement in cach ‘short conversation. Then read the question, and choose the best answer to that question. Remember that the best answer is one that shows agreement, 1. (woman) This homework is going to take forever. (man) Tl say! (narrator) What does the man mean? 2. (woman) A trip to the park might be (man) You can say that again! (narrator) What does the man mean? 3. (man) I think it's time t0 go home. (woman) So do I (narrator) What does the woman ‘mean? (A) Its going to take forever to get home. (B) It takes a long time to get from home to work. (© He and the woman have the same ‘opinion about the homework. (D) He needs to take the homework to class. (A) The woman should repeat what she said. (B) Its nice in the park at night. (© The woman should tell him about part of the trip. (D) He agrees about the trip to the park. (A) They dontt have time to do it. (B) She also thinks it’s a good idea to leave. (©) She thinks they are at home. (D) They are unsure about the time, TOEFL EXERCISE 6: In this exercise, listen carefully to the short conversation and question on the recording, and then. choose the best answer to the question. You should pay particular attention to expressions of agreement. C7 niow scan ne neconons at TOEFL Beast 6, 1. (A) She needs to check into the hospital. (B) She thinks the man should visit her (© She doesn't want to goto the hospital (D) She shares the man’s opinion. 2. (A) The man should repeat himself. (B) The prize was quite new. (C) She was also surprised. (D) The new surface was praised. 3. (A) The woman has a nice pet. (B) He agrees with the wovaan. (©) Abit of luck would be nice. (D) He should put the drinks on ice. 4. (A) They moved just after the baby ‘was born. (B) He saw that the woman was moving. (©) The movie was really good. (D) He shares the woman’ opinion. (continued on next page) LUSTENING COMPREHENSION 5. (A). She has over three classes, (B) She's glad to talk about the classes. (©) She'd like him to tell her about the classes. (D) She's also happy that the classes are finished. 6. (A) His ideas about sports are similar to the woman’. (B) They should watch a game on television together (C) He thinks he’s better at sports than the woman, (D) He doesnt like sports at all. 7. (A) She needs a straw for the coffee. (B) She agrees with the man, (©) She has been coughing strongly. (D) She is feeling strong today. 8. (A) It was possible to take a math class. (B) The matter could not possibly be discussed. (C) It was impossible to attend the math exam, (D) He has the same opinion asthe ‘woman, 9. (A) They always go out for dinner. (B) They should cook dinner outside. (C) She thinks they should go out, too. (D) She would like the man to prepare dinner tonight. 10, (A) He is in agreement about the teacher. (B) Her story was quite interesting. (©) He would like the woman to respect what he said. (D) The history teacher will give the same lecture again. TOEFL REVIEW EXERCISE (Skills 1-6): In this exercise, listen carefully to the short conversation and question on the recording, and then choose the best answer to the question, Qe ae 1. (A) Baking some pies. (B) Climbing some trees. (C) Having some dessert. (D) Picking some apples. 2. (A) There are thirty questions on the test. (B) About a half hour remains. (©) The test will begin soon. (D) Thinry students are taking the test. 3. (A) She agrees with the man, (B) They should visit their friend Bill. (C) It would be a good idea to play billiards. (D) The bills have already been paid. 4. (A) A florist, (B) A barber. (C) Achef. (D) A gardener. 5. (A) She fatled the test. (B) She's not exactly sure about the (C) Her statistics were quite exact. (D) She received a passing grade. 6. (A) Ina fastfood restaurant, (B) Ina grocery store, (©) Inan airport. () Ina service station, & 7. (A) Going down to the bottom of the lake. (B) Swimming in the lake, (© Riding a boat across the lake. (D) Taking a picture of the beautiful lake. 8. (A) He asked a number of questions. (B) He expressed his doubts about the situation. (©) He is an honorable man. (D) He said what he realy felt. USTENING PARTA 29 9. (A) He would like a cold drink, too. (8) The woman should repeat what she said. (C) He doesn't like the taste of the drink, (D) Its too cold to stop for a drink. 10. (A) He wants to have some water. (B) The grass is quite dry. (©) He's waiting for the loan to be approved. (D) He's going to mow the lawn this ‘week. 30 LUSTENING COMPREHENSION THE LISTENING PART B QUESTIONS ___ Part B of the Listening Comprehension section of the TOEFL test consists of two ong conversations, each followed by a number of questions. You will hear the ‘conversations and the questions on a recording; they are not written in your test book. You must choose the best answer to each question from the four choices that are written in your test book. ‘The conversations are often about some aspect of school (how difficult a class 4s, how to write a research paper, how to register for a course) or about general liv- ing (renting an apartment, playing sports, going to the bank). The conversations can also be about topics currently in the news in the United States (desalination of the water supply, recycling of used products, damage from a storm or some other type of natural phenomenon). Trample On the recording, you hear: (narrator) Questions 1 through 4. Listen to a conversation between two people who are decorating an ‘partment. (woman) Hey, Walt. Do you think you could help me hang these pictures on the wall? There are ony 1 of them. (man) Sure, Monica. Where do you want them to g0? (woman) I'd like the picture of the mountains over the fireplace, and If like the picture of my family ‘ver the sofa. What do you think? (man) think theyll ook fine there. FTow about if you hold the pictures while Ihomimer the nails ino the wall? (woman) Okay. Let's start with the picture of my family. Questions: 1, On the recording, you hear: (naator) What are the man and woman discussing? In your test book, you read: (A) Taking some pictures, (B) Hanging some plants, (C) Taking a trip to the mountains. (D) Putting some pictures on the wall. 2. On the recording, you hear: (narrator) How many pictures are there? In your test book, you read: (A) One. : B) Two. (©) Three. (D) Four USTENING PART B31 3. On the recording, you hear: (narrator) Whereis the picture ofthe woman’ family going? In your test book, you read: (A) Inthe fireplace. (B) Above the sofa. (©) Home with Walt. (©) To the top of the mountain. 4. On the recording, you hear: (narrator). What is Walt probably going to do nest? " Jn your test book, you read: (A) Sit on the sofa. () Photograph Monica’ family. (©) Hammer the nails into the wall (©) Climb the walls. The first question asks what the man and woman are discussing. Since the woman asks the man to help hang these pictures on the wall, the best answer to this ques- tion is answer (D), putting some pictures on the wall. The second question asks how many pictures there are, and the woman clearly says that there are two, so the best answer is answer (B). The third question asks where the family picture is going. The woman says that she would like the family picture over the sofa, so the best answer to this question is answer (B), above the sofa. The last question asks what Walt is probably going to do, Walt has suggested that he should hammer the nails into the wall, so the best answer is answer (C). STRATEGIES FOR THE LISTENING PART B QUESTIONS 1. Ifyou have the time, preview the answers to the Listening Part B questions. While you are looking at the answers, you should try to anticipute the questions for each of the groups of answers. 2. Listen carefully to the first line ofthe conversation. The fist line of the conversation often contains the main idea, subject, or topic ofthe conversation, and you will often be asked to answer such questions. 3. As you listen to the conversation, follow along with the answers in your test book, and try to determine the correct answers. Detail questions are generally ‘answered in order in the conversation, and the answers often sound the same as what is said on the recording, 4. You should guess even if you are not sure. Never leave any answers blank. 5. Use any remaining time to look ahead at the answers to the questions that follow. 32 USTENING COMPREHENSION The following skills will help you to implement these strategies in Part B of the Listening Comprehension section of the TOEFL test SKILL 7: THE QUESTIONS. It is very helpful to your ability to answer individual questions in Listening Part B if you can anticipate what the questions will be and listen specifically for the answers to those questions. ‘Example In your test book, you read: (A) On’Monday. (B) Next week. (©) Tomorrow. (D) After class. You try to anticipate the question: When will something happen? In this example, you can be quite certain that one of the questions will be about when something will happen. Since you are sure that this is one of the questions, you can listen carefully for clues that will give you the answer. This example shows that a helpful strategy therefore is to look briefly at the answers in the test book, before you actually hear the conversations on the recording, and try to determine the questions that you will be asked to answer. EXERCISE 7: Study te following answers and try to determine what the ques- tions will be. (You should note that perhaps you will only be able to predict part of a question, rather than the complete question.) If you cannot predict the question ina short period of time, then move on to the next group of answers. Question 1 has been answered for you. Cesc Na 20 Toate ey Pit tere. Oy Neeneioee Ee (A) What the assignment is. (B) How good the professor is. (©) The information in Chapter Six. (D) What questions they should ask the professor: 2. Question: (A) Six pages. (B) Six chapters. (C) Sixty pages. (D) The sixth chapter 10. - Question: - Question: /- Question: - Question: . Question: LUSTENING PART 833 Question: (A) Whether or not they should read the chapters. (B) Whether or not they should answer the questions. (©) Which chapters they should read (D) When the professor gave the assignment. (A) Tum in the assignment. (B) See the professor. (©) Gotoclass. (D) Talk to a classmate, - Question: (A) A bee stung him. (B) He saw some bees and hornets. (©) He was stung by a hornet. (D) He took some eggs from a nest. (A) To liberate bees. (B) To protect their nests. (©) To hatch their eggs. (D) To defend the park. (A) Ahomet’s nest. (B) Some bee's eggs. (©) A parked car (D) A swarm of bees. (A) To stay indoors. (B). To see where the hornet’ nest is located. (C) Not to walk in the same location as Greg. (D) To keep away from Greg. (A) The size of the campus. (B) The city bus system. (©) The length of time for each class. (D) The university bus system. Question: (A) The entire campus. (B) Part of the campus. (©) The campus and the city. (D) Only the off-campus areas. (continued on next page) 34 LUSTENING COMPREHENSION. 11. Question: (A) Nothing. (B) Three dollars. (©) A few cents. (D) Fifty cents. 12, Question: (@) Red. (B) Green. © Yellow. (D) Blue. Skit 8: THE TOPIC. ‘As you listen to each conversation in Listening Part B, you should be thinking about the topic (subject) or main idea for each conversation. Since the first one or two sentences generally give the topic, you should be asking yourself what the topic is while you are listening carefully to the first part of the conversation. Example (On the recording, you hear: (narrator) Listen to the conversation between two students. (man) What did you think of that history exam? (woman) That was the hardest exam Tve ever seen. (man) And it wasn't just hard! Te was long, too. ‘You think: The topic of conversation is a very long and difficult history EXEXC'SE 8; Listen to the first part of each of the conversations, and decide on the topic of each conversation. (ieee 1, What is the topic of Conversation 1? 2. What is the topic of Conversation 2? 3. What is the topic of Conversation 3? USTENING PART 35 Skit 9: THE ORDER OF THE ANSWERS. ‘The answers in Listening Part B are generally found in order in the conversation. ‘This means that as you listen to the conversation, you should be thinking about the answers to the questions in the order that they are listed in the test book. Example On the recording, you hear: In your test book, you read (same (narrator) Questions 1 and 2. Listen time): to two students on a 1. A) Hes sick, university eampus. (B) Hes lost (man) Can you help me? Tm lost. (©) Hes tired. (woman) Sure. Where are you trying (D) He's broke. 10 602 (man) Ihave a class in Stanfield 2. (A) Directly in front of them. Hall at 3:00. I thought I B) To the left. knew where Iwas going, (©) Quite nearby. but I guess I was wrong. (D) On the other side of campus. (woman) You certainly are lost. ‘Stanfield Hall is on the other ‘side of the university. Tre heading in that direction. Come on with me and I show you the way. (man) Thanks. You're a lifesaver. ‘On the recording, you hear: (narrator) 1. What problem does the mar have? 2, Where is Stanfield Hall? ‘When you read the answers to the first question, you can anticipate that the first question is about a man and some type of problcm he has. As you listen, you hear the man say that he is lost. Therefore, you can anticipate that the best answer to the first question will be answer (B). ‘When you read the answers to the second question, you can anticipate that the second question is going to ask where something is. In the conversation, the ‘woman explains that Stanfield Hall is on the other side of the university. Therefore, as you are listening you can anticipate that the correct answer to the second ques- tion is answer (D). 36 LISTENING COMPREHENSION EXERCISE 9: Listen to each complete conversation, and answer the questions that follow. C7 iow sean me reconoiis ar Berast 9 1. (A) What the assignment is. (B) How good the professor is. (©) The information in Chapter Six. (D) What questions they should ask the professor 2. (A) Six pages. (B) Six chapters. (©) Sixty pages. (D) The sixth chapter. 3. (A) Whether or not they should read the chapters. (B) Whether or not they should answer the questions. (©) Which chapters they should read. (D) When the professor gave the assignment. 4. (A) Tum in the assignment. (B) See the professor. (©) Goto class. (D) Talk to a classmate. 5. (A) Abee stung him. (B) He saw some bees and hornets. (©) He was stung by a hornet. (D) He took some eggs from a nest. 6, (A) To liberate bees. (B) To protect their nests. (C) To hatch eggs. (D) To defend the park. 7. (A) Ahornet’s nest. (B) Some bee's exes. (© A parked car. (D) A swarm of bees. 8. (A) Tostay indoors. (B) To see where the hornet’s nest is located. (© Not to walk in the same location as Greg. (D) To keep away from Greg, 9. (A) The size of the campus. (B) The city bus system. (C) The length of time for each class. (D) The university bus system. 10, (A) The entire campus. (B) Part of the campus. (©) The campus and the city. (D) Only the off-campus areas. 11, (A) Nothing, ®) Three dollars. ©) A few cents. () Fifty cents. 12, (A) Red. (B) Green. (©) Yellow. (©) Blue. USTENING PART B37 TOEFL EXERCISE (Skills 7-9): In this exercise, you will use all of the informa- tion that you learned in Skilis 7 through 9. OY iow exc rie neconons ar TOEFL Banas (Sas 7-9) 1. (A) Where the woman lives. (B) Where the man can find out the time. (©) Where the school is located. (D) Where the man can mail something. 2. (A) Several streets away. (B) Just over one block away. (C) Four miles away. (D) A great distance. 3. (A) For one hour. (B) For two hours. (©) For four hours. (D) For five hours. 4, (A) Go buy a package. (B) Go pick up his mail. (©) Go mail a package. (D) Get into his car. 5. (A) Froma textbook. (B) From the television. (C) From a magazine. (D) From lecture. 6. (A) How trees are grown in America. (B) How paper is made from trees. (C) Why paper is used so much. (D) The amount of paper that ‘Americans use. 7. (A) 50 tons. (B) 50 million tons. (© 85 million tons. (D) 850 million tons. 8. (A) Cut down more trees. (B) Use less paper. (©) Produce more paper. (D) Read more about the problem. 9. (A) The difficulty in finding some books. (B) How far itis to the library. (C) The amount of reading they must do, (©) How much they don't ike the ‘ook. 10, (A) Asspeed-reading class. (B) An American novels class. (©) Alibrary skills class. (D) Aclass in literary criticism. AL. (A) Some novels. (B) Some short stories. (© A textbook. (D) Some journal articles. 12. (A) Goto the library. (B) Head for class. (C) Return to the dorms. (D) Start reading. 38 LUSTENING COMPREHENSION THE LISTENING PART C QUESTIONS. Part C of the Listening Comprehension section of the TOEFL. test consists of three talks, each followed by a number of questions. You will hear the talks and the questions on a recording; they are not written in your test book. You must choose the best answer to each question from the four choices that are written in your test book. Like the conversations in Listening Part B, the talks are often about ‘some aspect of school life or topics currently in the news. It is also very common for the talks to be shortened versions of lectures from courses taught in Ameri- can colleges and universities. Example (On the recording, you hear: (narrator) Questions 1 through 4, Listen to a lecture in a history class. (woman) Sal, which today seems so plentiful to us, inthe past used to be a valuable commodity I the ‘ancient past in China, salt was used to make ‘coins, and in parts of Africa it was traded in lace of money. In the Roman Empire, soldiers fwere not paid in money but were instead paid in salt In fact, the English word "salary" which ‘means the amount of money that one earns, comes from the Latin root for "sal Questions: 1. On the recording, you hear: (narrator) What is the topic of the talk? In your test book, you read: (A) Valuable commodities. (B) Salt. (© Ancient China. () Money. 2. On the recording, you hear: (narrator) What was salt used for in China? In your test book, you read: (A) To spice food. (B) To build houses. (C) To make coins. (D) To locate Arica. UUSTENING PART C 3. On the recording, you hear: (narrator) What does “salary” mean in English? Tn your test book, you read: (A) Coins. (B) Earnings. (©) Soldiers. (©) Commodities. 4, On the recording, you hear: (narrator) What is the meaning of the root In your test book, you read: (a) Salt (B) Rome. (©) Money. (D) Trade. ‘The first question asks about the topic of the talk. The speaker begins with salt and continues to talk about it throughout the passage, so the best answer is answer (B). The second question asks about the use of salt in China. The speaker ‘says that in China, salt was used to make coins, so the best answer is answer (C). ‘The third question asks the meaning of salary. The speaker says that salary means the amount of money that one earns, so the best answer is answer (B). The last question asks about the meaning of the root sal. The speaker says that “salary”... comes from the Latin root for “salt,” so the best answer is answer (A). STRATEGIES FOR THE LISTENING PART C QUESTIONS 1. Ifyou have time, preview the answers to the Listening Part C questions. While | you are looking at the answers, you should try to anticipate the questions for each of the groups of answers. 2. Listen carefully to the first line of the talk. The first line of the talk often contains the main idea, subject, or topic of the talk, and you will often be asked this type of question. 3, As you listen to the talk, follow along with the answers in your test book and try to determine the correct answers. Detail questions are generally answered in order in the talk, and the answers often sound the same as what is said on the recording. 4, You should guess even if you are not sure. Never leave any answers blank. 5, Use any remaining time to look ahead at the answers to the questions that follow. ‘The following skills will help you to implement these strategies in Part C of the Listening Comprehension section of the TOEFL test. 39 40 LUSTENING COMPREHENSION Skit 10: THE QUESTIONS. Itis very helpful to your ability to answer individual questions in Listening Part C if you can anticipate what the questions will be and listen specifically for the answers to those questions (as you did in Listening Part B). Example In your test book, you read: (A) Fora week: (B) Since yesterday. (© Fortwo days. (D) Since 10:00 this morning. ‘You try to anticipate the question: How long has (something) been going on? Ih this example, you can be quite certain that one of the questions will be about how long something has been going on. Since you are sure that this is one of the questions, you can listen carefully for clues that will give you the answer. This ‘example shows that a helpful strategy in Listening Part C (just as in Listening Part B) therefore is to look briefly at the answers in the test book, before you actually hhear the talks on the recording, and try to determhine the questions that you will be asked to answer. EXERCISE 10: Study the following answers and try to determine what the ques- tions will be. (You should note that perhaps you will only be able to predict part of a question, rather than the complete question.) If you cannot predict the question in a short period of time, then.move on to the next group of answers. Question 1 has been answered for you. 1. Question: _What type of plant is tha? —__ (A) A beautiful plant. @) A poisonous plant. (©) A delicious plant. (©) A fast-growing plant. 2. Question: (A) In vegetable gardens. (B) Only in the United States, (©) In supermarkets. (D) In many different places. 3. Question: (A) Its leaves resemble parsley. (B) It grows next to carrots. (C) Its leaves are shaped like carrots. (D) It does not have roots. LISTENING PART C 4. Qoestions (A) The person may die. () The person may get lots of healthful nutrients. (©) The person may enjoy it and want more. (D) The person may become dangerous. 5. Question =e (A) Astory-witing contest. (B) A frog-catching contest. (©) A singing contest. (D) A frog-jumping contest. Cece. eee (A) Simy-three. (B). Two hundred. (©) Two thousand. (D) Forty thousand. 7. Question: a (@) One. (B) Two. (©) Three. (D) Four 8. Question: (A) The contest took place for years before Twain wrote about it. (B) Twain wrote about the contest while he was watching it for the frst time, (C) Twain went to see the contest many times during his lifetime. (D) ‘Twain wrote about the contest before it actually took place. 9. Question: (@) Astudent. (B) A professor. (© A bookstore clerk. (D) Alibrarian. 10. Question) Se ee (A) The place where students get ID cards. (B) The place where students can use computers, (C) The place where students check books out. (D) The place where students find books in the library. LL. Question; — @) Afee. (B) A student ID card. (©) Permission from the instructor. (D) A computer. (continued on next page) a 42 _USTENING COMPREHENSION 12 Question: (A) A few hours. (B) Two days. (C) Fourteen days. (D) Two months. ‘SkILL 11: THE TOPIC. As you listen tb each talk in Listening Part C, you should be thinking about the topic (subject) or main idea for each talk (as you did in Listening Part B). Since the first sentence is generally a topic sentence, you should be asking yourself what the topic is while you are listening carefully to the first part of the talk. Example (On the recording, you hear: (narrator) Listen to a talk a the start of a meeting (woman) Ta like to call this meeting to order now: This is the third monthly mecing of he scence cub this semester, and today we need to discuss the upcoming science fair. You think: The topic of the talk is a meeting of the science club to discuss the science fair. EXERCISE 11: Listen to the first part of each of the talks, and decide on the topic of each talk. OY Now scan me nconons ar Exencst 11. 1, What is the topic of Talk 1? 2. What is the topic of Talk 2? 3. What is the topic of Talk 3? LUSTENING PART C Skit 12: THE ORDER OF THE ANSWERS. The answers in Listening Part C are generally found in order in the talk (as they were in Listening Part B). This means that as you listen to the talk, you should be thinking about the answers to the questions in the order that they are listed in the test book. Example On the recording, you hear: In your test book, you read (same (narrator) Questions 1 through 3. time): Listen to a talk about cats. 1, (A) They shout. (woman) Many people are allergic 10 (B) They drive red cars. ‘cats. Ifthey come in contact (C) They sneeze, ‘with cats, they sneeze their (D) They-elose their eyes skin turns red, and their eves begin to burn. However, 2. (A) They often wear perfume. itis not only people who (B) They can have allergies. suffer from allergies. Cats (©) They dont ever suffer. also be allergic to (D) They like dust and pollen. pollen, dust, and perfumes, many of the same agents 3. (8) Nothing. that cause allergies in (B) Bathe it frequently. people. Perhaps your cat is (C) Putit outside. ‘sneezing and has watery () eyes. If you think that your cat has some allergies, a veterinarian can prescribe medication to help solve the problem. (On the recording, you hear: (narrator) 1. What happens to people who suffer from allergies? 2. What is meruioned about cats? 3. What can someone do swith a cat that has allergies? The first question asks what happens to people who suffer from allergies, The speaker says that they sneeze, so the best answer is answer (C). The second ques. tion asks what is mentioned about cats. The speaker says that cats may also be allergic, so the best answer is answer (B), they can have allergies. The third question asks what to do with a cat that has allergies. The speaker says that a veterinarian can prescribe medication, so the best answer is answer (D), give it medicine. “a LUSTENING COMPREHENSION EXERCISE 12: Listen to each complete talk, and answer the questions that follow. OY iow scan ne neconoaus ar Exencse 12. 1. (A) A beautiful plant. (B) A poisonous plant. (© A delicious plant. (D) A fast-growing plant. 2. (A) In vegetable gardens. (B) Only in the United States. (© In supermarkets. (D) In many different places. 3. (A) Its leaves resemble parsley. (B) It grows next to carrots. (©) Us leaves are shaped like carrots, (D) It does not have roots. 4, (A) The person may die, (B) The person may get lots of ‘healthful nutrients. (©) The person may enjoy it and ‘want more. (D) The person may become dangerous. 5. (A) A story-writing contest. (B) A frog-catching contest. (© A singing contest. (D) A frog-jumping contest, 6. (A) Sixty-three. (B) ‘Wo hundred. (C) Two thousand. (D) Forty thousand. 7. (A) One. (B) Two. (©) Three, (D) Four. 8, (A) The contest took place for years ‘before Twain wrote about it. (B) Twain wrote about the contest while he was watching it for the first time. (C) Twain went to see the contest ‘many times during his lifetime. (D) Twain wrote about the contest before it actually took place. 9. (A) Astudent. (B) A professor (©) Abookstore clerk. (D) A librarian. 10, (A) The place where students get ID cards. (B) The place where students can use ‘computers. (©) The place where students check: ‘books out. (D) The place where students find books in the library. 11. (A) A fee. (B) A student identification card. (C) Permission from the instructor. (D) Acomputer 12, (A) Afew hours. (B) Two days. (©) Fourteen days. (0) ‘wo months. USTENING PART C45, ‘TOEFL EXERCISE (Skills 10-12): In this exercise, you will use all of the infor- mation that you learned in Skills 10 through 12. O> iow sean me neconowis ar TOEFL Exenase (Sets 10-12). (A) An artist, (B) A circus performer (©) John Ringling. (D) A tour guide, . (A) Because he was from Sarasota, Florida. (B) Because he knew Rubens. (C) Because he started a circus. (D) Because he painted baroque-style paintings. . (A). Modern circus equipment. (B) Paintings by Rubens. (©) Aparade wagon. (D) A famous portrait of the Ringling brothers. (A) Enter the museum. (B)_ Go to the circus. (© Return to the bus. (D) Meet the Ringlings. (A) An assembly line. (B) Acar (©) Acompany. (D) An inventor. 5. (A) In 1908. @) In 1914, (©) In 1918, (D) In 1926. (A) Tewas faster. (B) Ie was more efficient. (C) It was more individualized. (D) Itwas cheaper 8, (A) It increased slowly. (B) Itincreased quickly. (©) Itremained about the same. (D) It decreased. 9. (A). Students who will soon graduate from the Psychology Department. (B) Professors in the Psychology Department. (©) Graduate students in the Psychology Department. (D) Graduate advisors. 10, (A) Whether to write a thesis or take (B) Whether to be graduate or ‘undergraduate students.” (©) Whether to graduate this year or next. (D) Whether or not to study. ‘psychology. 11, (A). Exams covering one or two hundred pages. (B) Exams about research. (C) Exams covering all material in the program. (D) Exams about recent developments in psychology. 12. (A) Today. (B) Soon. (©) Within six weeks. (D) Within half a year. 46 lo1lo1olalat TOEFL POST-TEST ‘SECTION 1 LISTENING COMPREHENSION “Time—approximately 35 minutes ncluding the reading of the directions for each part) In this section ofthe test, you will hve an opportunity to demonstrate your ability to understand conversations and tas in English. There are three parts to thie ection, with special directions foreach art. Answer all the questions onthe basis of what ie stated or implied by the speakers you hear. Do ‘ot take notes or write in your test book at any time. Do not turn the pages until you are told to do 0, Part A Directions: In Fart A you will hear short conversations between two people. After each conversation, ou will hear a question about the conversation. The conversations and questions will mot be repeated, ‘Afer you hear a question, read the four possible answers in your test book and choose the best answer ‘Then, on your answer sheet, find the mumber of the question and fil inthe space that corresponds to the leter ofthe answer you have chosen. Here is an example itn. orgies tema g © re) that x vai (roman) Oh i could have Been arse e Anarrator) What does the woman mean? In your test book, you will ead (A) The exam was really awful, (B) Teswas the worst exam she had ever seen, (©) Iteouldstt have been more dificult. () Aewasnt that ard, ‘You lear from the conversation that the man thought the exam was very difficult and thatthe ‘woman disagreed with the man, The best answer to the question, “What does the woman mean?” Is (D), “wasnt that hard." Therefore, the correct choice is (D) LUSTENING COMPREHENSION POST-TEST loljal L@ @ © ‘She doesn't want to goto class. [Art has her glasses. ‘The artist wl begin the portrait tonight. ‘The course starts this evening. Avvaiter: ‘Avbaker. neighbor ‘Atour guide. ‘She's inthe center ofthe shop. ‘She went shopping for @ new ear ‘She took her ear othe store. ‘She was driving him erazy. He led. Hes upset. He did not tell her his name. Hits always truthful. He believes that the woman has found ‘some good plces. Hi agrees that the prices are low. Hie woul like to have a chance to say ‘something. He thinks thst the worman is wrong ‘about the prices. Listen to him. Pay the check. ‘Attend the meeting. ‘Speake more leary: © © [Not moving inside. Playing in the rain, Not going out. Running bard. © © 8. (A) The courts have decided on a new Judge. (B) ‘The judge made himself available for ‘questions. (©) The judge decided on the issue. (D) The decision ebout the judge was finally ‘made. °. 13. 4. 1s. 16. Lol w ® © © wo co) LUSTENING COMPREHENSION POST-TEST 1o101 ‘She was unable to attend the chemistry clase ‘She couldn’ find the answer to the problem. ‘Chemistry class just finished ‘She has a problem with her chemistry teacher. Stay home. Watch television. Buy a new house. Gout Ina business office. Inan alrplane. Ina gymnasium. Ina classroom. ‘He fs probably swimming. He is poor. Nobody knows where he is. He should pot bein the pool. ‘Taking naps during the day. ‘Sleeping longer during the night. 7 Getting him red out before sleeping. Only sleeping a lide at night. ‘The chapter was dificult tread. ‘She didnt even ty to read the chapter. ‘She got through the chapter rater easily, ‘he wil ty to read the chapter later today. His garden is successful. ‘There ae alot of stones inthe garden, He enjoys the rocking char. eautifl weather is impossible. ‘She thinks the wedding is beautiful. She shares the man’s opinion ‘The red dress is incredible EE) a 101 1. 19. a. 24, w @ © © - (AY ® © © @) © o “ © o Arallroad conductor. ‘Abus driver. ‘A math teacher mechanic, ‘The sandwich needed some spices. ‘The lunch did not taste very good. ‘She had a delicious meal ‘She hardly tasted the sandwich, ‘She's no longer sick. ‘She has no feelings. er health i always good. Hr fels better than she does. ‘Watching the professor closely in class, Spending more tine working in his Staying epcbalog mor fe, ‘Talking to his professor 1s dificult for him to save money. He wants to purchase a home near the He goes to the beach often. Buying @ house is out of reach for him, Sally has many friends Hee doesnt understand anything about Sally. The stereo is not loud enough. He is going to turn the stereo off ‘The woman should turn snd face the stereo. ‘The woman doesn't want to hear the ‘music ee was not present during the overture. He was in over his head. He repeatedly expressed hie ‘appreciation, He thnks the present is overpriced 48 _USTENING COMPREHENSION POST-TEST 2s. 26, 2, 28, 29, 20, lolol w @ © © “ ® © o w @) © © In a bank, Ina hotel, Hanging the plctures on the wall, ‘Taking some photographs. Sitting loser tothe wal Visiting his hometown, ‘She doesn’t know who Calis. She already had a meeting with Cas), ‘She needs to tll Carl about the ‘esting. ‘She knows where Carl lives. His book was not really cheap. He bought a used tex. His book does not inelude the latest He did not know anything about the ‘entbooks. 1evas announced tha there would be pew teacher He wanted to give something tothe teacher ‘The instructor said that a test would be ‘ven. ‘The teacher returned the exams, There hae been alot of decay. ‘The government is decadent. ‘The government has decided to conduct ‘anew survey. ‘The population is counted every ten years, lolol 1olol Part B ‘Directions: In this part ofthe test, you will hear longer conversations, After each conversation, you ‘wil hear several questions. The conversations and questions will nt be repeated. lailaio1ol [After you hear a question, read the four possible answers in your test book and choose the best answer, ‘Then, on your answer sheet, find the number of the question and fil in the space that corresponds to the letter ofthe answer you have chosen. [Remember you are not allowed to take notes or write in your test book. Atone oclock. (B) Attwo ocock, (©) Atthree clock. () Atfour o'clock. 32. (A) Ski. (B) Read books on skiing (©) Bay siding equipment. (D) Plan ski ips He doesn't know how to ski. (B) He docent know where the meeting is. (©) He doesnt know what time the mesting (D) Hels afraid of skiing 34. (A) Leave on a sling tip. (B) Go with the woman to the meeting. (© Byonsome skis. (D) Give a lecture to the sk cub. 37. 38, 5. (A). rom the radio. (B) Fromabook. (© From the newspaper, (D) roma lecture (A) You can eat it (B) Tetastes lke fast food. (© Misinexpensive. (@) You cannot se it ) Paper (B) Fries. © com, (D) Burgers, () Beis not as good a5 paper (B) should nat be used for fas food. (©) Ie should be faster than paper (D) Iemight be healthier than the food. Pesaran LUSTENING COMPREHENSION POST-TEST 49 50 Part C Directions: In Part C of this section, you will hear several talks. After each talk, you wil hear some ‘questions. The talks and questions will not be repeated. ‘After you bear a question, you will read the four possible answers in your test book and choose the best answer Then, on your answer sheet, find the numberof the question and fil inthe space that corresponds tothe letier ofthe answer you have chosen, Here isan example. (On the recording, you wil hear feamatee) Listen oa insimetor ko hs cass about painting “Amis Grant Wood was a guiding force in the school of painting known as American ‘egionalist, a syle reflecting the distinctive characteristics ofa from rural areas of ie United States. Wood began drawing animals on the family farm atthe age of three, and ‘when he was thirty-eight one of his paintings received a remarkable amount of public notice and acclaim. This painting, called American Gothic, sa starkly simple depiction of serious couple staring directly out atthe viewer Nowlin os le goin, Semple Aner earn). Wht sect sini eo? g Inyo testo yw ea 8) Atm Aner nei g rer isi (© Ae tan asin he Sa Suse (©) Art from rural sections of America, ‘The best answer to the question, “What syle of painting is mown as American regionalist?” is (D), “Are from rural sections of America,” Therefore, the corectcholce is (D). [Now listen to another sample question. ‘Sample Answer (narrator). What isthe name of Wood's most successful painting? Jn your test book, you will read: (A) American (B) The Family Farm in lowe, (©) American Gothic. (D) A'Serious Couple e088 ‘The best answer tothe question, "What i the name of Wood!s most successful painting?” i (C), ‘American Gothie. Therefore, the correct choice is (C). Remember, you are not allowed to take notes or writ in your test book. @) [SETS the copegh perce Howe Semunersa at LISTENING COMPREHENSION POST-TEST ‘qnttontn poked Remon baa Lololotololoiol 2%. 40. a. 2 “4 1ol 1ol lolol ‘To the Grand Canyon. Toa jewelry show. ‘Toa deserted stone building. ‘To the Petrified Forest. ‘To the Grand Canyon. ‘To the Colorado River. ‘Toa tree house. ‘To the Petrified Forest. [thas fallen jn the river. thas turned to stone. thas grown larger. thas gotten softer ‘Swim in the river: ‘Walkthrough the tees Stay ata distance. ‘Take any petrified wood. Graduation requirements. School clothing. “The date of the December graduation ‘ceremony. ‘Which students willbe seniors. Read it File tin their personal files. Seudy it. Give it to an advisor 4s. 46. a. 48. 49. 50. (A) A university lass schedule. (B) “Acap and gown. (©) Ablue and gold fag. (D) A graduate school catalogue. (A) Any time before graduation. (B) Atthe beginning of the senior year (©) Before the end of December. (D) Inthe springtime. (B) Techniques of dog trainers. (©) The smells of various types of food. (D) How dog breeds are different. (A) Most have no sense of sme {(B) They areal unable to smell drugs. (C) They have equally good senses of smell, ©) Mort are quite untranable. ‘Thisis the end of Section 1. Stop work on Section 1. ‘Turn off the recording. ©Q0MCO8 ‘When you finish the test, you may do the following: + Tum to the Diagnostic Chart on pages 357-363, and circle the numbers of the questions that you missed. + Tum to the Progress Chart on page 353, and add your score to the chart. LUSTENING COMPREHENSION POST-TEST 51 When you finish the test, you may do the following: « Turn to the Diagnostic Chart on pages 357-363, and circle the numbers of the questions that you missed. * Turn to the Progress Chart on page 353, and add your score to the chart. ‘STRUCTURE AND WRITTEN EXPRESSION PRE-TEST 6 62 STRUCTURE AND WRITTEN EXPRESSION The second section of the TOEFL test is the Structure and Written Expression section. This section consists of forty questions (some tests may be longer). You have twenty-five minutes to complete the forty questions in this section. ‘There are two types of questions in the Structure and Written Expression sec- tion of the TOEFL test: 1, Structure (questions 1-15) consists of fifteen sentences in which part of the sentence has been replaced with a blank. Each sentence is followed by four answer choices. You must choose the answer that completes the sentence in a grammatically correct way. 2. Written Expression (questions 16-40) consists of twenty-five sentences in which four words or groups of words have been underlined. You must choose the underlined word or group of words that is not correct. GENERAL STRATEGIES |. Be familiar with the directions. The directions on every TOEFL test are the same, so itis not necessary to spend time reading the directions carefully when you take the test. You should be completely familiar with the directions before the day of the test. 2, Begin with questions 1 through 15. Anticipate that questions 1 through 5 will be ‘the easiest. Anticipate that questions 11 through 15 will be the most difficult. Do, ‘ot spend too much time on questions 11 through 15, There will be easier ‘questions that come later, 3, Continue with questions 16 through 40. Anticipate that questions 16 through 20 willbe the easiest. Anticipate that questions 36 throu'zh 40 will be the most difficult. Do not spend too much time on questions 36 through 40. ‘time, return to questions 11 through 15. You should spend extra time ns 11, through 15 only after you spend all the time that you want on the easier questions. '5. Never leave any questions blank on your answer sheet. Even if you are not sure of the correct response, you should answer the question, There Is no penalty for ‘guessing. STRUCTURE THE STRUCTURE QUESTIONS Questions 1 through 15 in the Structure and Written Expression section of the TOEFL test measure your knowledge of the correct structure of English sentences. The questions in this section are multiple-choice questions in which you must choose the letter of the answer that best completes the sentence. Example —— sreeted me enthusiastically at the front door (A) Parental (B) If (©) My friends () Them In this example, you should notice immediately that the sentence has a verb, greeted, and that the verb needs a subject. Answers (A), (B), and (D) are incorrect, because parental, if, and them are not subjects. The correct answer is answer (C).. STRATEGIES FOR THE STRUCTURE QUESTIONS 1. First study the sentence. Your purpose isto determine what is needed to complete the sentence correctly. 2. Then study each answer based on how well it completes the sentence. Eliminate answers that do not complete the sentence correctly. 3, Do not try to eliminate incorrect answers by looking only at the answers. The Incorrect answers are generally correct by themselves, The incorrect answers are generally incorrect only when used to complete the sentence. 4, Never leave any answers blank. Be sure to answer each question even if you are unsure of the correct response. 5. Do not spend too much time on the Structure questions. Be sure to leave ‘adequate time for the Written Expression questions. ‘The following skills will help you to implement these strategies in the Structure section of the TOEFL test. 3 64 STRUCTURE AND WRITTEN EXPRESSION. Skit. 1: SUBJECTS AND VERBS ‘You know that a sentence in English should have a subject and a verb. The most common types of problems that you will encounter in the Structure section of the TOEFL test are related to subjects and verbs; perhaps the sentence is missing either the subject, or the verb, or both; perhaps the sentence has an extra subject or verb. Example 1 —— was ringing continuotisly for hours. (A) Loudly (B) In the morning (©) The phone (D) The bells In this example, you should notice immediately that there is a verb, was ringing, but there is no subject. Answer (C) is the best answer because it isa singular sub- Ject that agrees with the singular verb was ringing. Answer (A), loudly, and answer (B), in the morning, are not subjects, so they are not correct. Although answer (D), bells, could be a subject, it is not correct because bells is plural and it does not agree with the singular verb was ringing. Example 11 ‘Newspapers ___ every morning and every evening. (A) delivery (B) are delivered © ontime (D) regularly In this example, you should notice immediately that the sentence has a subject, newspapers, but that there is no verb. Because answer (B), are delivered, is a verb, it is the best answer. Answers (A), (C), and (D) are not verbs, so they are not correct. Example 111 ‘The plane __ landing at the airport in five minutes. ) itis (B) it really is (C) is descending (D) will be “Wihs semtence has a subject, plane and has part of.a vetb, landing; to be comect, some form of the verb be is needed to make the verb camleles ARWLS (2) tied STRUCTURE (B) are incorrect because the sentence already has a subject, plane, and does not need the extra subject it. Answer (C) is incorrect because descending is an extra part of a verb that is unnecessary because of landing. Answer (D) is the best answer; will be together with landing is a complete verb. The following chart outlines.the key information that you should remember * about subjects and verbs. EXERCISE 1: Underline the subjects once and the verbs twice in each of the fol- lowing sentences. Then indicate if the sentences are correct (C) or incorrect (I). abs ales 1. My bestfriend always helpful with problems. 2. The bus schedule has changed since last week. 3. Accidentally dropped the glass on the floor. 4. The customer paying the clerk for the clothes. 5. The professor handed the syllabus to the students. 6. Bach day practiced the piano for hours. 7: The basketball player tossed the ball into the hoop. 8, The new stident in the class very talkative and friendly. 9. Walking with the children to school. 10. The whales headed south for the winter, 6 (66 STRUCTURE AND WRITTEN EXPRESSION ‘SKILL 2: OBJECTS OF PREPOSITIONS An object of a preposition is a noun or a pronoun that comes after a preposition such as in, at, of, 10, by, behind, and on to form a prepositional phrase. ‘The trip (to the island) (on Saturday) will last (for three hours). This sentence contains three objects of prepositions. Island is the object of the Preposition 10; Saturday is the object of the preposition on; hours is the object of the preposition for. ‘An object of a preposition can cause confusion in the Structure section of the ‘TOEFL test because it can be mistaken for the subject of a sentence. Example ‘To Mike ____was a big surprise. (A) really (B) the party (©) funny (D) when In this example, you should look first for the subject and the verb. You should notice the verb was and should also notice that there is no subject. Do not think that Mike is the subject; Mike is the object of the preposition fo, and one noun cannot be both a subject and an object at the same-time. Because a subject is needed in this sentence, answer (B), the party is the best answer. Answers (A), (C), and (D) are not correct because they cannot be subjects. The following chart outlines the key information that you should remember about objects of prepositions. NOTE: A ist of prepositions and exercises to practice recognizing ‘these prepositions can be found In Appendie 8 a the back ofthe ‘text. You may want to complete these exercises before continuing with Bxerle 2, smmuctune 67 EXERCISE 2: Each of the following sentences contains one or more prepositional phrases. Underline the subjects once and the verbs twice. Circle the prepositional phrases that comne before the verb. Then indicate if the sentences are correct (C) or incorrect (1). 1, The name @fibe bab) GRIRE AAD ig ack —L. 2. Gy the next meeting)@f the class) 3. The directions to the exercise on page 20 unclear. need to turn in the papers. —— 4. Because of the heavy rain throughout the night, the walkways are muddy. 5. During the week eat lunch in the school cafeteria. 6 In the morning after the concert was tired, _—— 7. Inthe summer the trip to the mountains is our favorite trip. _— & Ina box on the top shelf of the cabinet in the hallway of the house, 9. With her purse in her hand ran through the door. 10, At 1:00 in the morning the alarm clock on the table beside the bed rang. EXERCISE (Skills 1-2): Underline the subjects once and the verbs twice. Circle the prepositional phrases that come before the verb. Then indicate if the sentences are correct (C) or incorrect (1). 1. During the meeting in the office discussed the schedule. 2. The doctor gave the patient a prescription. ___ 3. The tall evergreen trees along the road. —— 4 The watch in the jewelry box needs a new battery. 5. Pleasantly greets everyone in all the offices every morning. 6, Inthe office of the building across the street from the park on the corner. 7. The dishes in the sink really need to be washed as soon as possible. 8. Ina moment of worry about the problem with the cash in the account. ‘STRUCTURE AND WRITTEN EXPRESSION 9, The plane from New York circling the airport. —— 10. Ona regular basis the plants in the boxes under the window in the kitchen are watered and fed. TOEFL EXERCISE (Skills 1-2): Choose the letter of the word or group of words that best completes the sentence. 1, Mark Twain ___ the years after the Civil War the “Gilded Age.” (a) called (B) calling (©) he called (D) his calls 2, Barly ___toes instead of hooves on their feet. (A) horses (B) had horses (©) horses had (©) horses having B grow close to the ground in the short Aretic summer, (A) Above tundra plants (B) Tundra plants. (© Tundra plants are found (D) For tundra plants 4, In 1867, __ Alaska from the Russians for $7.2 million. (A) purchased the United States (B) to purchase the United States (C) the United States’ purchase of (D) the United States purchased 5, Between 1725 and 1750, New England witnessed an increase in the specialization of. (A) occupations (B) occupies (© they occupied (D) it occupied them 6. The large carotid artery __ to the ‘main parts of the brain. (A) carrying blood (B) blood is carried (©) cartes blood @) blood carries 7, _—iadio as the first practical ‘stem of wireless telegraph. (A) Marconi’s development (@) ‘The development by Marconi (© Developing Marcon! ©) Marconi developed {8 In 1975, the first successful space probe to —__ beginning to send information back to Earth. (A) Venus (B) Venus the (© Venus was (D) Venus it was 9. The two biggest resort __ Arkansas are Hot Springs and Eureka Springs. @ in @) towns in © towns are (©) towns are in 10. NASAs Lyndon B. Johnson Space Center —_ control center for the Mercury, Gemini, and Apollo space flights, (A) itwas at the (B) it was the (©) was the (©) the STRUCTURE ‘Skit 3: PRESENT PARTICIPLES. Present participles can cause confusion in the Structure section of the TOEFL test because a present participle can be either an adjective or a part of the verb. A present participle is the -ing form of the verb. It is part of the verb when itis pre- ceded by some form of the verb be. ‘The train is arriving at the station now. VERB In this sentence, arriving is part of the verb because it is accompanied by is. A present participle is an adjective when it is not accompanied by some form of the verb be. ‘The train arriving at the station now is an hour late. In this sentence, arriving is an adjective and not part of the verb because it is not accompanied by some form of be. The verb in this sentence is is. The following example shows how a present participle can be confused with the verb in the Structure section of the TOEFL test. Example The film @) now ®) is © it @) was appearing at the local theater is my favorite. In this example, if you look at only the first words of the sentence, it appears that film is the subject and appearing is part of the verb. If you think that appearing is part of the verb, you might choose answer (B), is, or answer (D), was, to complete the verb. However, these two answers are incorrect because appearing is not part of the verb. sou should recognize that appearing is a participial adjective rather than a verb because there is another verb in the sentence, is. In this sentence, there is a complete subject, film, and a complete verb, is, so this sentence does not need another subject or verb. The best answer to this question is answer (A). ‘The following chart outlines the key information you should remember about present participles. abscea a NEC eee ° 70 STRUCTURE AND WRITTEN EXPRESSION EXERCISE 3: Each of the following sentences contains one or more present par- ticiples. Underline the subjects once and the verbs twice. Circle the present par- ticiples, and label them as adjectives or verbs. Then indicate if the sentences are correct (C) or incorrect (1). —C__ 1. The Gying) baby needs to be picked up. ADS te or —L_ 2. The clothes are(Iying)on the floor should go into the washing machine. ene air —— 3. The waitress bringing the steaming soup to the waiting diners. 4. Most ofthe striking workers are walking the picket line. —— 5. Forher birthday, the child is getting a talking doll. ——_ 6. The setting sun creating a rainbow of colors in the sky. 7. The ship is sailing to Mexico fs leaving tonight. —— 8. The letters needing immediate answers are on the desk. —— 9. The boring class just ending a few minutes ago. —— 10. The fast-moving clouds are bringing freezing rain to the area. SKILL :, PAST PARTICIPLES Past participles can cause confusion in the Structure section of the TOEFL test because a past participle can be either an adjective or a part of the verb. The past participle is the form of the verb that appears with have or be. It often ends in -ed, but there are also many irregular past participles in English. ‘The mailman has left a letter in the mailbox. ‘VERB. ‘The classes were taught by Professor Smith. VERB In the first sentence, the past participle left is part of the verb because it is accom- panied by has. In the second sentence, the past participle ‘aught is part of the verb because it is accompanied by were. STRUCTURE ‘A past participle is an adjective when it is not accompanied by some form of Be or have. ‘The letter left in the mailbox was for me. ‘ADSECTIVE ‘The classes taught by Professor Smith were very interesting. ‘ADIECTIVE, In the first sentence, left is an adjective rather than a verb because it is not accom- panied by a form of be or have (and there is a verb, was, later in the sentence). In the second sentence, taught is an adjective rather than a verb because it is not accompanied by a form of be or have (and there is a verb, were, later in the sen- tence). ‘The following example shows how a past participle can be confused with the verb in the Structure section of the TOEFL test. Example ‘The bread ____ baked this morning smelled delicious. (A) has (B) was © it () just In this example, if you look only at the first few words of the sentence, it appears that bread is the subject and baked is either a complete verb or a past participle that needs a helping verb. But if you look further in the sentence, you will see the verb smelled. You will then recognize that baked is a participial adjective and is therefore not part of the verb. Answers (A) and (B) are incorrect because baked is ‘an adjective and does not need a helping verb such as has or was. Answer (C) is incorrect because there is no need for the subject it. Answer (D) is the best answer to this question. ‘The following chart outlines the key information that you should remember about past participles. Gagzernal eens n ‘STRUCTURE AND WRITTEN EXPRESSION. EXERCISE 4: Each of the following sentences contains one or more past partici- ples. Underline the subjects once and the verbs twice. Circle the past participles, and label them as adjectives or verbs. Then indicate if the sentences are correct (©) or incorrect (1). LL 1. The food is Gerved) in this restaurant is delicious. ‘VERB —C__ 2, The plane landed on the @eserted) runway. = ‘a0. —— 3 the unexpected guests arrived just at dinnertime. —— 4. The courses are listed in the catalogue are required courses. —— 5. The teacher found the lost exam. —— 6 The small apartment very crowded and disorganized. —— 1. The photographs developed yesterday showed Sam and his friends. —— 8. The locked drawer contained the unworn jewels —— 9. The tree was blowa over in the storm was cut into logs —— 10. The students registered in this course are listed on that sheet of paper. EXERCISE (Skills 3-4): Each of the following sentences contains one or more participles. Underline the subjects once and the verbs twice. Circle the participles, and label them as adjectives or verbs. Then indicate if the sentences are correct, (O or incorrect (1). —— 1. Ourhosts are serving drinks on the tiled patio. — 2 The tired woman taking a much needed nap. ——_ 3. The letters were sent on Monday arrived on Wednesday. 4. The winners deserved the big prize. ‘5. The plants are growing inthe garden need a lot of water. —— 6. The shining stars lit up the darkened sky. 17. The driver rapidly increased the speed of the racing car. —— 8 The excited children trying to build a snowman in the falling snow. STRUCTURE 73 9. ‘The students are completing the course will graduate in June. 10. ‘The dissatisfied customer is returning the broken toaster to the store. TOEFL EXERCISE (Skills 3-4): Choose the letter of the word or group of words that best completes the sentence. ‘The first ___ appeared during the last period of the dinosaurs’ reign. (A) flowers are plants {B) plants have flowers (©) plants flowers (D) flowering plants ‘The eaitliest medicines __ from piants of various sorts. (A) obtaining (B) they obtained (©) were obtained () they were obtained ‘Simple sails were made from canvas —— ower a frame. @) astretch (B) stretched (©) was stretched (D) it was stretched |. Pluto's moon, Charon, ina slightly elliptical path around the planet, (A) moving (B) is moving (© it was moving (D) in its movement 5. Techniques of breath control form an essential part of any. improve the voice. program to (D) training 6. Robert B. Lee __ the Confederate 7, 10. army to General Grant in 1865 at the Appomattox Courthouse. (A) surrendered (B) he surrendered (©) surrendering (D) surrender ‘The pituitary gland, the brain, releases hormones to control other glands. (A) found below (B) itis found below (©) its foundation below (D) finds itself below . At around two years of age, many . Multinational companies children regularly produce sentences —— three or four words. (A) are containing (B) containing (C) contain (D) contains it {increasingly important to employ internationally acceptable brand (A) finding (B) are finding (©) they find (©) they are finding ‘The comea is located under the conjunctiva, on___ of the eye. (A) the partis exposed (B) exposed the part (©) the exposed part (D) exposes the part 74 “STRUCTURE AND WRITTEN EXPRESSION TOEFL REVIEW EXERCISE (Skills 1-4): Choose the letter of the word or group of words that best completes the sentence. first settled the Hawaiian Islands between A.D. 300 and 750. (A) The Polynesians (B) The Polynesians arrived (©) Because of the Polynesians (D) Itwas the Polynesians In 1066, a bright comet ____in the sky attracted much attention, (A) was appearing (B) appears (©) itappeared (D) appearing |. In some daguerreotype cameras, —— through a hole in the back of the box. (A) the object's view (B) the object was viewed (C) from the view of the object, (D) viewed the object }- In the Stone Age, stone tools —__ with other rock materials. (A) polishing (B) they polished (©) for polish (D) were polished ‘The first steamship to cross the Atlantic Savannah, in 1819. (A) was the (B) itwas the (©) the (D) init the 6. The Earth's plates meet each other at ccracks in the Barth ___ faults. (A) were called (B) calls (©) called (D) it was called 7. The first plant-like organisms probably in the sea, perhaps 3 billion years ago. (A) life (B) living (©) lived (D) it was li 8 8, In male pattern baldness, —_ strongly influences thé degree of hair loss. (A) heredity (B) inherited (©) inherits (D) heredity has 9, In Watch the Skies, Curtis Peebles ‘attempt to explain America’s belief in flying saucers. (A) makes a fascinating (B) making a fascinating (©) fascination with making (D) fascination made a 10, The irregular coastline of __a succession of bays and inlets, with the hook of the Cape Cod peninsula in the southeast. (A) Massachusetts (B) Massachusetts is (C) Massachusetts itis (D) Massachusetts on STRUCTURE ‘Skit, 5: COORDINATE CONNECTORS. Many sentences in English have more than one clause. (A clause is a group of words containing a subject and a verb.) When you have two clauses in an Eng- lish sentence, you must connect the two clauses correctly. One way to connect two clauses is to use and, but, or, or so between the clauses. ‘The sun vas shining, and the sky was blu. “The sky was blu, bu it was very cold. 1 may rin tonight, or it may be clear 1 was raining outside, so I took my umbrella. In each of these examples, there are two clauses that are correctly joined with a coordinate connector—and, but, or, or so—and a comma (,). The following example shows how this sentence pattern could be tested in the ‘Structure section of the TOEFL test. ‘Example forgot my coat, __ I got very cold. (A) then (B) so (©) later (D) asa result In this example, you should notice quickly that there are two clauses, I forgot my coat and J got very cold. This sentence needs a connector to join the two clauses. Then, later, and as a result are not connectors, so answers (A), (C), and (D) are not correct. The best answer is answer (B) because so can connect two clauses in this manner. The following chart lists the coordinate connectors and the sentence pattern used with them. 8 76 STRUCTURE AND WRITTEN EXPRESSION EXERCISE 5: Each of the following sentences contains more than one clause. Underline the subjects once and the verbs twice. Circle the connectors. Then indi- cate if the sentences are correct (C) or incorrect (I). -C_ 1, The lawn needs water every day, @F)it will turn brown. —L_ 2. The book was not long, (Bad) it difficult to read. ——_ 3: It was raining, so decided not to go camping. —— 4. The material has been cut, and the pieces have been sewn together. —— 5: The patient took all the medicine, he did not feel much better. —— 6. The bill must be paid immediately, or the electricity will be turned off. —— 7. The furnace broke so the house got quite cold. —— _& The dress did not cost too much, but the quality it seemed excellent. —— 9. The leaves kept falling off the trees, and the boys kept raking them up, but the ‘yard was still covered. —— 10. The mail carrier has already delivered the mail, so the letter is not going to arrive today, it probably will arrive tomorrow. ‘Skit 6: ADVERB CLAUSE CONNECTORS Sentences with adverb clauses have two basic patterns in English, Study the clauses and connectors in the following sentences: He is tired because he has been working so hard. Because he has been working so hard, he is tired. In each of these examples, there are two clauses: he is tired and he has been work. ing so hard. The clause he has been working so hard is an adverb clause that is introduced with the connector because. In the first example, the connector because ‘comes in the middle of the sentence, and no comma (,) is used. In the second example, the connector because comes at the beginning of the sentence. In this pattern, when the connector comes at the beginning of the sentence, a comma (,) is required in the middle of the sentence. The following example shows how this sentence pattern could be tested in the Structure section of the TOEFL test. STRUCTURE ‘Example ___ arrived at the library, he started to work immediately. (A) The student (B) When (©) He (D) After the student In this example, you should recognize easily that the verb arrived needs a subject. ‘There is also another clause, he started to work immediately. If you choose answer (A) or answer (C), you will have a subject for the verb'arrived, but you will not hhave a connector to join the two clauses. Because you need a connector to join ‘two clauses, answers (A) and (C) are incorrect. Answer (B) is incorrect because there is no subject for the verb arrived. Answer (D) is the best answer because there is a subject, student, for the verb, arrived, and there is a connector, after, to join the two clauses. ‘The following chart lists common adverb connectors and the sentence patterns used with them. EXERCISE 6: Each of the following sentences contains more than one clause. ‘Underline the subjects once and the verbs twice. Circle the connectors. Then indi- cate if the sentences are correct (C) or incorrect (1). he plane circled the airport, it Janded on the main runway 12, The registration process took many hours Gines) the lines so Jong. 3. This type of medicine can be helpful, it can also have some bad side effects. 78 STRUCTURE AND WRITTEN EXPRESSION —— 4. The waves were amazingly high when the storm hit the coastal town, 5. We need to get a new car whether is on sale or not. —— 6 Iustas the bread came out of the oven, while a wonderful aroma filled the Kitchen. —— 7. Everyone has spent time unpacking boxes since the family moved into the new house. —— & Although the area is a desert many plants bloom there in the springtime. —— 9. The drivers on the freeway drove slowly and carefully while the rain was falling heavily because they did not want to have an accident. —— 10. Ifyou plan carefully before you take a trip, will have a much better time because the small details will not cause problems. EXERCISE (Skills 5-6): Each of the following sentences contains more than one clause. Underline the subjects once and the verbs twice. Circle the connectors. Then indicate if the sentences are correct (C) or incorrect (1). —— 1. The lawyer presented a strong case, but the client was still found guilty. —— 2. After the children read some stories before they went to bed. —— 3. The report needed to be completed, the workers stayed Iate every night for a week. " —— -4. Ifyou do not turn on the lights, you will trip in the dark. —— 5. Athick fog came rolling in, so planes unable to land. —— 6. Allof the shoes are on sale until the current stock is gone. —— 1. The ship leaving the dock even though some passengers were not on board. —— The outline must be tuned in to the teacher a week before the paper is due, ‘and must approve it. —— 9. Because the food was cold when it was served the diners sent it back to the Kitchen. —— 10. You should slow down while you are driving, or the police will pull your car stmuctuRE 79 TOEFL EXERCISE (Skills 5-6): Choose the letter of the word or group of words that best completes the sentence. 1. A spacecraft is freed from friction —— launched into space. A) it B) itis (©) after is (D) after itis —— with their surroundings, or they hide in crevices for protection. (A) Lobsters (B) Lobsters blend (©) Lobsters blending (D) Because lobsters blend ‘a ball-and-socket joint, the elbow is a simple hinge joint. (A) While the shoulder (B) While the shoulder is (©) The shoulder is (D) The shoulder 4, Acarhas several sections with ‘moving parts, of those parts is essential. (A) g00d lubrication (B) well lubricated (©) and good lubrication (D) and well lubricated 5, Bears cannot see well eyes. (A) bears have (B) because having (©) because they have (D) because of bears small at the Isthmus of Panama, so animals were able to migrate between North and South America. (A) Aland bridge existed (B) When a land bridge existed (©) Aland bridge (D) With a land bridge 2 10. mostly made of granite, it also contains some human-made materials, (A) The Empire State Building (B) The Empire State Building is (© Although the Empire State Building is (D) Although the Empire State Building is built . Pressure differences make the ‘eardrum vibrate the ear. (A) enters the sound waves (B) as sound waves, (©) sound waves enter (D) as sound waves enter . An optical microscope magnifies as ‘uch as 2,000 times, but an electron microscope ——as much as a million times, (A) magnifying (B) itmagnifies (©) can magnify (D) magnify it If scientific estimates are accurate, ‘with the Earth about 20,000 ‘years ago. () the Cafon Diablo meteorite collided (B) the collision of the Cation Diablo meteorite (©) the Cafion Diablo meteorit colliding . (D) colliding the Cafion Diablo ‘meteorite ‘80 STRUCTURE AND WRITTEN EXPRESSION TOEFL REVIEW EXERCISE (Skills 1-6): Choose the letter of the word or group of words that best completes the sentence. 1 of the Pueblo Indians centered on intensive agriculture. (A) The economic activity (B) Because the economic activity (C) The economy was active (D) When the economic activity In popular terminology, any long snowstorm with is called a blizzard, (A) the amount of wind is large (B) a large amount of wind (© itis very windy (D) very windy . Nuclear power can be produced by fusion, ___ produced by fission. (A) itcan also be (B) itcan also (©) and it can also be (D) and it can also ——. igneous rocks may be changed into gneisses. (A) The temperature is high (B) If the temperature is high (©) High temperatures (D) Ifhigh temperature In 1905, Henry Flagler ___ his plans to extend his Florida East Coast Railway out across the sea to Key West. (A) itwas announced (B) Announcement (©) the announcement of (D) announced ‘The sound ____from a vibrating ‘object will be high or low depending con the number of vibrations, (A) comes (B) itis coming (©) is coming (D) coming 7. During the late 1880s, urban streetcars were electrified through large motors. (A) they used (B) used (©) the use of (D) when they used 8. almost 274 square miles, but 96 percent of the park is under water. (A) Although Biscayne National Park encompasses (B) Biscayne National Park encompasses (©) Biscayne National Park ‘encompassing (D) Biscayne National Park 9. Legislation __ in 1916 and 1917 gave the Wilson administration authority to intervene in the national economy if it proved necessary. (A) itwas passed (B) was passed (©) passed (D) passes, 10. Because a family of birds set up housekeeping in Joel Chandler Harris's mailtox when the birds were in need of a place :9 stay, __ the ‘Wren's Nest. (A) the home is named (B) so the home is named (© naming the home (D) the home’s name STRUCTURE 81 ‘Skit. 7; NOUN CLAUSE CONNECTORS. Announ clause is a clause that functions as a noun; because the noun clause func- tions as a noun, it can be used in a sentence as an object of a verb (if it follows a verb) or an object of a preposition (if it follows a preposition). Study the clauses and connectors in the following sentences. {don't know [hey he sald such things. ‘NOUN CLAUSE AS OBIECT OF VERB ‘am thinking about [why he said such things] NOUN CLAUSE AS OBECT OF PREPOSITION In the first example, there are two clauses, I don't know and he said such things. ‘These two clauses are joined with the connector why, Why changes the clause he sate ‘such things into a noun clause which functions as the object of the verb don't now. In the second example, the two clauses I am thinking and he said such things are also joined by the connector why. Why changes the clause he said such things into a noun clause which functions as the object of the preposition about. ‘The following example shows how these sentence patterns could be tested in the Structure section of the TOEFL test. Example ‘The citizens worry about is doing. (A) what the government (B) the government (©) what (D) what the government it In this example, the sentence contains the main subject and verb, the citizens worry, and it also contains an additional verb, is doing. The sentence needs a sub- ject for the verb is doing and a connector to join the two clauses. The best answer is answer (A) because it has the connector what and the subject government. ‘Answer (B) is incorrect because it does not have a connector. Answer (C) is incor- rect because it does not have a subject for is doing, Answer (D) is incorrect because it has two subjects for is doing. a

You might also like